Você está na página 1de 4

Problem 1. Let f be a function defined on a closed domain.

Show that f is continuous if


and only if the inverse image of every closed set is a closed set.
Solution. Now assume f is continuous. Let D be a closed set, then we need to prove that
f 1 (D) is also closed. If we have a sequence xn x0 in which xn f 1 (D), then we are
done if we can show that x0 f 1 (D) as well. Since f is continuous, we have a convergent
sequence limn f (xn ) = f (x0 ) = y. Notice that we used the property that x0 is within the
domain of f , which is closed as preassumed, so f (x0 ) makes sense. Therefore we completed
the first part if we can recall that D is closed, which leads y no where else to go but staying
inside D.
Now assume for any closed D, the preimage f 1 (D) is also closed. Now choose xn x0
in the domain of f , which automatically gives us an x0 in the domain as well because the
domain is assumed to be closed. Now assume f (xn ) does not converge to f (x0 ), which is
equivalent to say that |f (xni ) f (x0 )| > for some fixed > 0 and for some subsequence
xni . Now consider the closure of this subsequence f (xni ), which, by assumption, does NOT
include f (x0 ). By our initial assumption, we know that f 1 (xni ) is also closed. However,
this is impossible, because the point x0 , which happens to be a limit point of f 1 (xni ), is
not contained by this set. So we know f (xn ) must be convergent to f (x0 ).

Problem 2. Let A be the set defined by the equations f1 (x) = 0, . . . , fn (x) = 0, where
f1 , . . . , fn are continuous functions defined on the whole line. Show that A is closed. Must
A be compact
Solution. We know the solution is

n
\

f 1 (0)

i=1

which is closed by the continuity of fi . However they may not be compact. For example, we
can let f1 fn 0.

Problem 5. Show that the function f (x) = x on [0, 1] for 0 < 1 satisfies a Holder
condition of order for 0 < but not for >
Solution. Apparently it suffices if we can show
(x + h) x
h

is bounded for x, x + h [0, 1], h > 0, for 0 < . Now since we know that
h h . Therefore we reduce the problem to estimating the following fraction
(x + h) x
h
Now, if h x, then we have
(x + h) x
(h + h)

2 2
h
h
Consequently, if h < x, then we have, by mean value theorem
(x + h) x
h1

h
(x + h0 )1
in which h < x. Now it is easy to see it is bounded by

h1
1
x1

Therefore the function is of Holder condition with constant M = 2. For < case, just let
x = 0, and h 0.

Problem 7. Give an example of a continuous function with domain R1 such that the
inverse image of a compact set is not compact
Solution. f (x) 0

Problem 8. Give an example of a continuous function with Domain R1 such that the
image of a closed set is not closed.
Solution. f (x) = tan1 (x)

Problem 11. If f is continuous on R1 , is it necessarily true that f (lim supn xn ) =


lim supn f (xn )?
Solution. No. Choose any sequence an 1 and bn 2, then shuffle these two sequences
into a new one cn , which has lim supn cn = 1. Then choose f (x) = x.

Problem 3. If the domain of a continuous function is an interval, show that the image is
an interval. Give example where the image is an open interval
Solution. Let a be the greatest lower bound of the range of f on some interval I(could
be open, closed, or half-closed-half-open), and b be the lease upper bound. Then by the
intermedia value theorem, any interval (a0 , b0 ) such that a < a0 < b0 < b will be within f (I).

Therefore we know that f (I) =]a, b[, here the special term ][ means an interval, could be
open, closed, or half-open-half-closed.

Problem 4. If a continuous function on an interval takes only a finite set of values, show
that the function is constant.
Solution. This is a direct corollary of the previous problem, corresponding to the case when
a = b.

Problem 5. Suppose f and g both satisfy a Lipschitz condition on an interval. Show that
f + g also satisfy a Lipschitz condition.
Solution. This is trivial by triangular inequality, say |f (x) + g(x) f (y) g(y)| |f (x)
f (y)| + |g(x) g(y)|. However, for an arbitrary function, let us say, h(x), to satisfy Lipschitz
condition, we mean there exists an M , such that |h(x) h(y)| < M |x y|. What is
important is that this M varies among different functions, and even different region where
x, y are. Therefore you should use M1 for f , and M2 for g in this question. And the final
bound will be
|f (x) + g(x) f (y) g(y)| |f (x) f (y)| + |g(x) g(y)| (M1 + M2 )|x y|.

Problem 7. Let f be a monotone function on an interval. Show that if the image of f is
an interval, then f is continuous. Give an example of a non-monotone funcion on an interval
whose image is an interval but that is not continuous.
Solution. If f fails to be continuous, then by Theorem 4.2.6, there must be a point x0 in
the domain of f such that limxx0 f (x) < limxx+0 f (x). By the monotonicity of f , there
must be a hole in the range of f , which contradicates with initial assumption. So f is
continuous.
Now for the example, consider
(
x + 1, x [1, 0]
f (x) =
x + 2, x [0, 1]

Problem 8.
If f = p + g where p is a polynomial of odd degree and g is a bounded
ocntinuous function on the line. Show that there is at least one solution of f (x) = 0.
Solution. Since p is a polynomial of odd degree, we know p() = , and p() = ,
therefore so is true for p + g as g is a bounded function. Now the proof follows if one recalls
the intermediat value theorem.

Problem 15. Give an example of a function on R1 that assumes its sup and inf on every
compact interval and yet is not continuous.
Solution.

(
1,
f (x) =
0,

x is ratioanl
x is irrational
3

Você também pode gostar